1answer.
Ask question
Login Signup
Ask question
All categories
  • English
  • Mathematics
  • Social Studies
  • Business
  • History
  • Health
  • Geography
  • Biology
  • Physics
  • Chemistry
  • Computers and Technology
  • Arts
  • World Languages
  • Spanish
  • French
  • German
  • Advanced Placement (AP)
  • SAT
  • Medicine
  • Law
  • Engineering
Vitek1552 [10]
3 years ago
12

5x² + 4x - 20 = 0 Solve this using the quadratic formula

Mathematics
1 answer:
Natali [406]3 years ago
8 0

Here's the answer, using the quadratic formula

You might be interested in
What is this equal<br> how can I solve similar trigonometric integrals like this one
Angelina_Jolie [31]

Answer:

ln|sec θ + tan θ| + C

Step-by-step explanation:

The integrals of basic trig functions are:

∫ sin θ dθ = -cos θ + C

∫ cos θ dθ = sin θ + C

∫ csc θ dθ = -ln|csc θ + cot θ| + C

∫ sec θ dθ = ln|sec θ + tan θ| + C

∫ tan θ dθ = -ln|cos θ| + C

∫ cot θ dθ = ln|sin θ| + C

The integral of sec θ can be proven by multiplying and dividing by sec θ + tan θ, then using ∫ du/u = ln|u| + C.

∫ sec θ dθ

∫ sec θ (sec θ + tan θ) / (sec θ + tan θ) dθ

∫ (sec² θ + sec θ tan θ) / (sec θ + tan θ) dθ

ln|sec θ + tan θ| + C

3 0
3 years ago
Nina wants to buy a bicycle thats costs $150. she has saved 6 percent of the total cost of the bicycle.how much money has nina s
notka56 [123]
The understanding one needs to work with percentages is:
1/100 = 1% = 0.01
Also
6/100 = 6% = 0.06
also
25/100=1/4=0.25=25%

To find a spesific share of a total, we can multiply the perectage with the total, like this: Total * 6% or in a way easier to work with: Total * 0.06 or (Total * 6)/100

If you have a calculator i would recommend using the decimals, as they are one of the easiest to type in - and remember once you get the hang of it.

To find the money Nina has saved, you do the following.
150*0.06=9$
Nina has saved 9 dollars.

The remaining percent she needs, is
100-6=94%

She still needs money for 94% of the bike.
8 0
3 years ago
5,049 +48=105
Alborosie
<h2>Solution (a) :-</h2>

Let the cost of the less expensive comic book be x .

Then :-

1284 - 11x = x

Using this equation , we can find the cost of the less expensive comic book .

Price of the less expensive book :-

1284 - 11x = x

11x + x = 1284

12x = 1284

x =  \frac{1284}{12}

x = 107

Cost of the less expensive book is 107 .

Therefore , the epice of the less expensive book = $ 107 .

<h2>Solution (b) :-</h2>

Cost of the more expensive book = 11x

Which means :-

1284 - x = 11x

11x + x = 1284

12x = 1284

x = 107

11x = 11 \times 107

= 1177

Therefore , the price of the more expensive book = $ 1177 .

4 0
3 years ago
Help me please thank you
Harman [31]

Answer: \\ j(m) = {2}^{m - 1} \Rightarrow j(4) =  {2}^{4 - 1}  =  {2}^{3}  = 8

6 0
3 years ago
If the legs of a right triangle are 6 cm and 5 cm find the area of the triangle​
Ilia_Sergeevich [38]

Answer: 15 cm²

Step-by-step explanation: To find the area of a triangle, start with the formula for the area of a triangle which is shown below.

Area = \frac{1}{2}bh

In this triangle, the base is 6 centimeters and the height is 5 centimeters. Now, plugging into the formula, we have (\frac{1}{2})(6 cm)(5 cm).

Now, it doesn't matter which order we multiply.

So we can begin by multiplying 1/2 times 6 cm and notice that the 2 in 1/2 and the 6 cross cancel to 1 and 3. So we're left with 3 cm.

Now, (3 cm)(5cm) is 15 cm².

So the area of the triangle is 15 cm².

6 0
3 years ago
Other questions:
  • Which of the following sets is equal to {1,2,3...}?
    5·1 answer
  • Confused plz help! will mark!!
    11·2 answers
  • A baker made 60 muffins for a cafe. By noon, 45% of muffins were sold. How many muffins were sold by noon?
    11·1 answer
  • Find the volume of the solid bounded by the plane z = 0 and the paraboloid z = 1 - x2 - y2. SOLUTION If we put z = 0 in the equa
    7·1 answer
  • Pls help<br><br> Solve for m<br><br> a=f/m
    7·1 answer
  • Which angles!!! Need answers 40% of my grade please and thank you
    14·1 answer
  • Mathwiz wya?? pic below i need help asap
    12·1 answer
  • a student needs to fill a tank with 9 liters of waters how many 1/3 liters of water will it take to fill the tank
    15·2 answers
  • 17. Two tanks are similar in shape. The capacity of the tanks are 1,000,000 litres and 512, 000 liters respectively
    10·2 answers
  • subset to real numbers, and as you can see are not at all connected to the rest of numbers (and integers).
    10·1 answer
Add answer
Login
Not registered? Fast signup
Signup
Login Signup
Ask question!